23.11.2012 Views

Solutions

Solutions

Solutions

SHOW MORE
SHOW LESS

You also want an ePaper? Increase the reach of your titles

YUMPU automatically turns print PDFs into web optimized ePapers that Google loves.

Problem Set 4<br />

MATH 778P, Fall 2008, Cooper<br />

Expiration: Wednesday, December 10<br />

You are awarded up to 15 points for each problem, 5 points for submitting<br />

solutions in L ATEX, and 5 points per solution that is used for the answer key.<br />

1. Let G = (V, E) be the graph whose vertices are all 7 n vectors of length n<br />

over Z7, in which two vertices are adjacent iff they differ in precisely one<br />

coordinate. Let U ⊂ V be a set of 7 n−1 vertices of G, and let W be the<br />

set of all vertices of G whose distance from U exceeds (c + 2) √ n, where<br />

c > 0 is a constant. Prove that |W | ≤ 7 n e −c2 /2 .<br />

Solution (Mark Walters): Fix c > 0 and define ɛ = e −c2 /2 . Consider<br />

{0, 1, 2, 3, 4, 5, 6} n as the underlying probability space, where all points are<br />

equally likely. For y ∈ {0, 1, 2, 3, 4, 5, 6} n set<br />

X(y) = min ρ(x, y)<br />

x∈U<br />

where ρ is defined to be the Hamming metric on {0, 1, 2, 3, 4, 5, 6} n . Let<br />

X0, X1, . . . , Xn = X be the martingale given by exposing one coordinate<br />

of {0, 1, 2, 3, 4, 5, 6} n at a time. The Lipschitz condition holds for X: If<br />

y, y ′ differ in just one coordinate then |X(y) − X(y ′ )| ≤ 1. Thus, with<br />

µ = E[X], Azuma’s inequality yields<br />

But<br />

so µ ≤ 2 √ n. Thus<br />

and<br />

Pr[X < µ − 2 √ n] < e −22 /2 1<br />

= ,<br />

e2 Pr[X > µ + c √ n] < e −c2 /2 = ɛ.<br />

Pr[X = 0] = |U|7 −n = 1<br />

7<br />

Pr[X > (c + 2) √ n] < ɛ<br />

1<br />

> ,<br />

e2 |B(U, (c + 2) √ n)| = 7 n Pr[X ≤ (c + 2) √ n] ≥ 7 n (1 − ɛ).<br />

It follows that |W | ≤ 7 n − 7 n (1 − ɛ) = 7 n ɛ.<br />

1


2. Let σ be a random permutation of [n], and fix a permutation τ of [k]. Recall<br />

that, for a set I ∈ � � [n]<br />

k , we say that “the pattern τ occurs in σ on I” if σ|I<br />

is order-isomorphic to τ. Denote by Xτ (σ) the number of sets I ∈ � � [n]<br />

k<br />

for which τ occurs in σ on I. Show that<br />

��<br />

���<br />

Pr X τ (σ) − 1<br />

k! ·<br />

� �� � �� �<br />

n ��� 2 n<br />

−(1 + o(1))ɛ n<br />

≥ ɛ < exp<br />

k k<br />

2k4 �<br />

.<br />

Solution (Aaron Dutle):<br />

Note that E[Xτ (σ)] = 1<br />

� � n<br />

k! k<br />

Consider the function Xτ (σ) and the martingale given by at step i revealing<br />

the location of the number i in σ. Note that given two partially<br />

revealed permutations, the next revealed number could on one extreme<br />

be completely unusable in finding τ in σ, or on the other, be usable as<br />

any of the k symbols of τ and with any choice of k − 1 elements from the<br />

remaining n − 1. Thus our martingale satisfies<br />

� �<br />

n − 1<br />

|Xi+1 − Xi| ≤ k<br />

k − 1<br />

So if we define a new martingale Y by Yi = Xi<br />

k( n−1 then the martingale<br />

k−1)<br />

Y0, Y1, . . . Yn is actually Lipschitz. Set µ = E[Y ] = Y0, and Y = Yn.<br />

Now we note that<br />

��<br />

���<br />

Pr Xτ (σ) − 1<br />

� �� � ��<br />

n ��� n<br />

≥ ɛ<br />

k! k k<br />

��<br />

����<br />

Xτ (σ)<br />

= Pr<br />

k � � �<br />

1 n<br />

k! k � n−1 −<br />

k−1 k � �<br />

� � � n<br />

� k � n−1 � ≥ ɛ<br />

�<br />

k−1 k � �<br />

� n−1<br />

k−1<br />

�<br />

= Pr |Y − µ| ≥ ɛ n<br />

k2 �<br />

�<br />

= Pr<br />

by Azuma’s inequality.<br />

Noting that<br />

�<br />

2 exp − ɛ2n 2k4 �<br />

|Y − µ| ≥ ɛ√ n<br />

k 2<br />

� �<br />

√<br />

n < 2 exp − ɛ2n 2k4 �<br />

�<br />

= exp ln(2) − ɛ2n 2k4 � � �<br />

= exp − 1 − 2k4 ln(2)<br />

ɛ2 � 2 ɛ n<br />

n 2k4 �<br />

and that k is fixed, this is<br />

� 2 −(1 + o(1))ɛ n<br />

exp<br />

2k4 �<br />

,<br />

the desired bound.


3. Show that, with high probability, G(n, 1/2) contains a path of length n(1/2+<br />

o(1)).<br />

Solution (Wei-Tian Li): Here we apply the Theorem 8.5.1. Set p =<br />

[(log n)/n]ω(n), where ω(n) → ∞ arbitrary slowly. Then in G(n, p) almost<br />

always deg(x) ∼ (n − 1)p for all vertices x. Let ω(n) = n/2 log n,<br />

then p = 1<br />

n<br />

2 and hence δ(G) > 2 with high probability. By the fact that if<br />

δ(G) > k, then there exists a path of length k in G, so with high probability<br />

G(n, 1/2) contains a path of length n(1/2 + o(1)).<br />

4. Prove that, for every ɛ > 0 there is some n0 = n0(ɛ) so that for every n > n0,<br />

there is a graph on n vertices containing every graph on k ≤ (2 − ɛ) log 2 n<br />

vertices as an induced subgraph.<br />

Solution (Xing Peng): Here, we want to use the general form of Janson’s<br />

inequality. The statement is following.<br />

Let I be the index set and {Bi}i∈I be a collection of events. Set XBi<br />

be the indicator variable of Bi and X = X . Set ∼ be the dependence<br />

i∈IBi<br />

graph. If µ = E[X], ∆ = �<br />

�<br />

Pr[Bi ∧ Bj] and δ = max Pr[Bj]. Then<br />

Pr[ �<br />

i∈I<br />

i∼j<br />

i<br />

i∼j<br />

Bi] ≤ exp(−min{ µ2 µ µ<br />

, ,<br />

8∆ 2 6δ }).<br />

For any natural number n, take a graph G(n, 1<br />

2 ). For a fixed graph H<br />

on k = (2 − ɛ) log2 n vertices, Let the index set be � � [n]<br />

k and BS be the<br />

event that H is isomorphic to the subgraph induced by some vertex set<br />

S ∈ � � [n]<br />

k . Then<br />

Pr[BS] = 1<br />

� � n<br />

k , µ =<br />

2) 2) ,<br />

and<br />

∆ =<br />

� � k−1<br />

n �<br />

k<br />

i=2<br />

� n−k<br />

k−i<br />

2 (k<br />

�� k<br />

i<br />

�<br />

2 2(k 2)−( i , δ =<br />

2)<br />

2 (k<br />

k−1 �<br />

i=2<br />

� n−k<br />

k−i<br />

Then 8∆<br />

µ 2 = 8 �k−1 i=2 g(i), where g(i) is defined in Section 4.5. By the proof<br />

of Theorem 4.5.1, we have 8∆<br />

µ 2 < k<br />

µ2 n<br />

n and then 8∆ > k . Simply estimation,<br />

µ n<br />

2 > k , here we need k = (2 − ɛ) log2 n and we can not omit ɛ. Moreover,<br />

µ n<br />

6δ > k . Hence,<br />

Pr[ �<br />

Bs] ≤ e −n/k .<br />

S∈( [n]<br />

k )<br />

For this H, let AH be the event that H is not a induced subgraph of<br />

G(n, 1<br />

2 ) and An be the event that there is some graph on k vertices which<br />

is not a induced subgraph of G(n, 1<br />

2 ). Then<br />

Pr[AH] < e −n/k .<br />

2 (k<br />

�� k<br />

i<br />

2)<br />

�<br />

.


and<br />

Pr[An] < 2 (k2)<br />

−n/k k<br />

e < 2 2 /2 −n/k k −n/k 1<br />

e < n e <<br />

nα Here, α > 1 and n sufficiently large.<br />

The Borel-Cantelli Lemma applies, alway all An fail for n sufficiently large.<br />

Thus, there is some n0 so that for every n > n0, there is a graph on n<br />

vertices containing every graph on k vertices as an induced subgraph.<br />

5. Find a threshold function for the property: G(n, p) contains at least n/6<br />

pairwise vertex disjoint triangles.<br />

Solution (Aaron Dutle):<br />

Claim that r(n) = n −2/3 is a threshold function for G(n, p) to contain n/6<br />

vertex disjoint triangles.<br />

Suppose that p(n) ≪ n −2/3 .<br />

If H is the graph made of n/6 vertex disjoint triangles, note that for a<br />

particular choice of the of 3-sets of vertices of the triangles, the probability<br />

that H occurs with these as the triangles in G(n, p) is p n/2 , since n/2 is<br />

the number of edges of H.<br />

If we consider making this choice by choosing three vertices for the first<br />

triangle, then choosing three for the second, then the third, etc. until we<br />

have n/6 of them, noting that there are then (n/6)! ways that we can<br />

choose the same set, we see there are<br />

� � n<br />

3,3,...,3,n/2<br />

(n/6)!<br />

such possible choices. So if X denotes the number of times H occurs in<br />

the graph, the expectation of X is<br />

E[X] =<br />

� n �<br />

3,3,...,3,n/2<br />

(n/6)!<br />

p n/2<br />

Rewriting this in an equivalent form, and then estimating using � �<br />

n n<br />

e <<br />

n! < nn we have<br />

E[X] =<br />

≤<br />

n!<br />

6n/6 � � � �<br />

n n p<br />

2 ! 6 ! n/2<br />

n n<br />

6n/6 � �<br />

n n/2 � � p<br />

n n/6<br />

2e 6e<br />

n/2<br />

= n n/3 2 n/2 e 2n/3 p n/2


Since p ≪ n−2/3 , eventually p <<br />

is<br />

< n n/3 2 n/2 e 2n/3<br />

�<br />

1<br />

8e4/3n2/3 1<br />

8e 4/3 n 2/3 after which the quantity above<br />

� n/2<br />

= 2 −n = o(1)<br />

Hence X = 0 almost surely, proving one direction of the threshold function.<br />

For the opposite direction, suppose that p(n) ≫ n −2/3 . To make the proof<br />

go easier, note that if p(n) > ln(n)n −2/3 we can randomly remove edges<br />

from our G(n, p) to reduce to the case p(n) = ln(n)n −2/3 . So we can<br />

assume that p(n) ≤ ln(n)n −2/3 .<br />

Consider a subset of the vertices of G(n, p) of size n/2. For each set of<br />

three vertices in this set, we let Bi be the event that the three vertices are<br />

a triangle. The expected number of triangles is then<br />

� �<br />

n/2<br />

µ = E[X] = p<br />

3<br />

3<br />

We’re going to use Janson’s inequality, so we compute ∆ = �<br />

i∼j Pr[Bi ∧<br />

Bj]. We choose a triangle, choose an edge of it, and choose a third vertex<br />

to count all of the i ∼ j. The probability of each of these is p 5 , giving<br />

� �<br />

n/2<br />

∆ = 3(n/2 − 3)p<br />

3<br />

5 .<br />

When applying Janson’s inequality, we get Pr �� �<br />

−µ+∆/2<br />

Bi < e<br />

Since this is the probability that a single choice of n/2 of the vertices<br />

contains no triangle, and the probability for any other choice of an n/2<br />

subset would be the same, conclude that � � �� � n<br />

n/2 Pr Bi is the number of<br />

n/2 subsets of the vertices that have no triangle. Estimating this, we find<br />

<<br />

� �<br />

n<br />

�� �<br />

Pr Bi<br />

n/2<br />

� �n/2 ne<br />

e<br />

n/2<br />

−µ+∆/2<br />

< e n e −µ+∆/2 = e n−µ+∆/2<br />

We want this quantity to go to 0 as n → ∞, so we need that the exponent<br />

is going to −∞.<br />

Since we know p < ln(n)n −2/3 , we can approximate<br />

n − µ + ∆/2


� �<br />

n/2<br />

= n − p<br />

3<br />

3 � �<br />

n/2<br />

+ 3(n/2 − 3)p<br />

3<br />

5<br />

< n − n3<br />

216e 3 p3 + n4<br />

64 p5<br />

≤ n − n3<br />

10 4 p3 + (ln(n))5<br />

n 1/3<br />

Since p ≫ n −2/3 , eventually p > 10 5/3 n −2/3 , so this quantity is<br />

which clearly goes to −∞.<br />

Thus � � �� � n<br />

n/2 Pr Bi = o(1).<br />

< n − 10n + (ln(n))5<br />

n 1/3<br />

= −9n + (ln(n))5<br />

n 1/3<br />

So with high probability every n/2 sized subset of G(n, p) has a triangle.<br />

To find n/6 vertex disjoint triangles, we take them greedily, choosing an<br />

n/2 sized subset, taking its triangle, choosing another n/2 subset avoiding<br />

our chosen triangle, and so on. With high probability, we only run out of<br />

n/2 subsets to choose when our triangles use n/2 of the vertices, which is<br />

when we have n/6 triangles chosen.<br />

Thus n −2/3 is a threshold function for G(n, p) having n/6 vertex disjoint<br />

triangles.<br />

6. Let G = (V, E) be a graph with chromatic number χ(G) = 1000. Let<br />

U ⊂ V be a random subset of V chosen uniformly among all 2 |V | subsets<br />

of V . Let H = G[U] be the induced subgraph of G on U. Prove that<br />

Pr[χ(H) ≤ 400] < 1<br />

100 .<br />

Solution (Yiting Yang): Let L(U) be the chromatic number of G[U].<br />

Let X0, X1, . . . , Xm be the martingale on L by exposing a color class each<br />

time. Xm(U) = L(U).<br />

Let U and U ′ be two subsets of V which differ only in one color class. Then<br />

|L(U ′ ) − L(U)| ≤ 1, which implies L satisfied the Lipschitz Condition. By<br />

Theorem 7.4.2, we have<br />

where µ = E[L].<br />

P r[L < µ − λ √ m] < e −λ2 /2 ,<br />

Recalling E[L] ≥ 500 in Homework 2, we have


P r[L[U] < 400] = P r[L[U] < 500 − √ 10 √ 1000]<br />

≤ P r[L[U] < µ − √ 10 √ m] < e −(√ 10) 2 /2<br />

= e −5 < 1<br />

100 .<br />

7. Show that there exists a binary string S of length at most c2n log n/n so<br />

that, for every w ∈ {0, 1} n , there is a length n subword of S identical to<br />

w except in at most one coordinate. (We say that w1w2 · · · wk−1wk is a<br />

subword of S1S2 · · · SN−1SN if there is some j so that Sj+i = wi for all<br />

1 ≤ i ≤ k.) Hint: Take a random string. How many w’s are missed? Tack<br />

them onto the end of the string.<br />

Solution (Mark Walters): Let w ∈ {0, 1} n and randomly select a<br />

string S of length 2n log n<br />

n . Let Ai be the event that the length n subword<br />

beginning at the ith coordinate of S is identical to w except in at most<br />

one coordinate. Note that<br />

Pr[Ai] =<br />

n + 1<br />

2 n<br />

because there are n choices for the “free” coordinate and of course one<br />

additional subword that didn’t require a “free” coordinate. Also, note<br />

that<br />

µ = �<br />

Pr[Ai] = (|S| − n + 1)<br />

i∈I<br />

n + 1<br />

2 n<br />

where I = {1, 2, . . . , |S| − n + 1}. Now, in the event that Ai and Aj are<br />

dependent events for i < j, we need to assess the amount of overlap, i.e.,<br />

the value of j − i, in order to calculate ∆. It’s obvious that the smallest<br />

overlap is 1 while the largest overlap is n − 1. If j − i = k then we get<br />

Pr[Ai ∧ Aj] ≤ n2 + k 2 + (2n − 1)(1 − k)<br />

2 2n−k<br />

because there are 2n − k coordinates involved and we have to allow that<br />

at least one of them is a “free” coordinate. So, we get<br />

∆ = �<br />

Pr[Ai ∧ Aj] =<br />

i∼j<br />

|S|−n+1 �<br />

|S|<br />

i=1<br />

i=1<br />

k=1<br />

n−1 �<br />

2<br />

k=1<br />

Pr[Ai ∧ Aj]<br />

� n−1 � n<br />

≤ 2<br />

2 + k2 + (2n − 1)(1 − k)<br />

22n−k |S|<br />

�<br />

<<br />

i=1<br />

�<br />

2 k<br />

2 −2n+2 n 3<br />

n−1<br />

k=1


|S|<br />

�<br />

= 2 −2n+2 n 3 (2 n − 1)<br />

i=1<br />

|S|<br />

�<br />

< 2 −n+2 n 3<br />

i=1<br />

= |S|2 −n+2 n 3<br />

Now plugging into The Janson Inequality we get<br />

� �<br />

�<br />

Pr ≤ exp � − (|S| − n + 1) n2 −n+1 + � |S|2 −n+2� /2 �<br />

i∈I<br />

Ai<br />

= exp � (−|S|n + n 2 − n)2 −n+1 + |S|2 −n+1�<br />

= exp � |S|2 −n+1 (1 − n) + (n 2 − n)2 −n+1�<br />

� n 2 log n<br />

= exp<br />

n<br />

�<br />

2 log n<br />

= exp<br />

n − 2 log n + n2 − n<br />

2n−1 �<br />

−2 log n<br />

→ e<br />

= n −2<br />

2 −n+1 (1 − n) + (n 2 − n)2 −n+1<br />

In other words, given a word w ∈ {0, 1} n , the probability that there does<br />

not exist a length n subword of S identical to w except in at most one<br />

coordinate is at most n −2 . Since there are 2 n possible words w ∈ {0, 1} n ,<br />

we need to tack on 2 n n −2 words of length n. In other words, we need to<br />

tack on 2n<br />

n many coordinates to S to form a new string ¯ S, which has fewer<br />

than 2 2n log n<br />

n<br />

coordinates. Since we are not even doubling the size of S,<br />

the size of ¯ S is c2 n log n/n and now for each w ∈ {0, 1} n there exists a<br />

length n subword of ¯ S identical to w except in at most one coordinate.<br />

8. Let G = G(n, p) with p = [(3 log n)/n2 ] 1/3 . Show that, with high probability,<br />

every pair of vertices in G is connected by a path of length 3. (Note<br />

that a path of length three has four vertices.)<br />

Solution (Bill Kay): First, we calculate the probability that a given<br />

pair of vertices has a path of length 3 between them. If � � n<br />

2 is o(P r[a<br />

given pair of vertices do not have a path of length 3) , we can apply the<br />

union bound. Pick two vertices at random, call them x0 and x3. For<br />

counting purposes, it is convenient to think of this path as going from<br />

x0, x1, x2, x3. Now, there are n − 2 ways to pick x1 and then n − 3<br />

ways to pick x2. Also, each of the edges has to be in the graph. If X<br />

is the random variable that counts the number of paths between x0 and<br />

x3, E[X] = µ = (n − 2)(n − 3)p3 . To calculate ∆, simply notice that two<br />

paths can only intersect in one place; If they intersect in the edge between<br />

x1, x2 then the other edges are fixed. We first count the number of ways<br />


two paths could intersect in the edge (x0, x1), and by symmetry we get<br />

how many ways two paths could intersect in the edge (x2, x3). First, there<br />

are n − 2 ways to choose the common (x1). Then, there are � � n−3<br />

2 ways<br />

to pick the middle edges. The last two edges are now fixed. Each edge<br />

has to be in, so the calculation gives ∆ = 2(n − 2) � � n−3 5<br />

2 p . We now apply<br />

Jansen’s inequality. The probability that none of the paths are in the<br />

graph is bounded above by e −(n−2)(n−3)p3 +(n−2)( n−3<br />

2 )p 5<br />

. Plugging in the<br />

proposed value for p, a little work gives:<br />

e −(n−2)(n−3)p3 +(n−2)( n−3<br />

2 )p 5<br />

= e −(n−2)(n−3)([(3 log n)/n2 ] 1/3 ) 3 +(n−2)( n−3<br />

2 )([(3 log n)/n 2 ] 1/3 ) 5<br />

≤ e −n2 ([(3 log n)/n 2 ])+n 3 ([(3 log n)/n 2 ] 5/3 )<br />

= e<br />

3 log n<br />

−3 log n+ 1<br />

n 3<br />

−3 log n+o(1)<br />

= e<br />

Clearly, � � n − log n<br />

2 = o(e ), so the union bound can be applied as desired.

Hooray! Your file is uploaded and ready to be published.

Saved successfully!

Ooh no, something went wrong!